Beweisen, dass isometrieerhaltende Exzision dem Töten ähnlich ist?

(Wenn Sie denken, dass dies zB nicht gut ausgedrückt ist, haben Sie die Bitte um Hilfe bereits verstanden.)

Satz : Gegeben eine Mannigfaltigkeit M mit einer Metrik ausgestattet G und mindestens eine nicht-triviale Isometrie besitzt ϕ generiert durch ein Killing-Feld K , für M ' = M W , Wo W ist eine offene Teilmenge in M ϕ : M ' M ist eine Isometrie genau dann, wenn P W ¯ , T P ( P ) M ' ist parallel zu K ( P ) .

Korollar : Die Symmetrien von W Bestimmen Sie die Isometrien von M ' : wenn Tangenten an W ¯ sind in K ich überall tangential zu W ¯ , K ich bleibt ein Killing Field von M ' .


Wenn nicht wirklich falsch, kann dies bekannt/trivial sein, aber ich kann keinen Beweis finden - und ich werde durch mangelndes Wissen und Notation daran gehindert, einen zu konstruieren (daher ist die Aussage möglicherweise auch nicht sehr gut; Bedingungen für die Mannigfaltigkeit fehlen, Zum Beispiel).

Nachdem ich im spezifischen Kontext der zeitähnlichen Killing Fields des Minkowski-Raums (Lorentzsche Metrik) darüber nachgedacht hatte, schien der oben angegebene allgemeine Fall (Riemmanian oder Lorentzian) plausibel, aber ein Beweis des Spezialfalls ist das, was ich wirklich brauche.

Skizzen . Das Theorem besagt, dass, wenn eine Region aus einer Mannigfaltigkeit herausgeschnitten wird, wenn die Grenze dieser Region den integralen Kurven eines Killing-Feldes der ursprünglichen Mannigfaltigkeit folgt, dieses Feld ein Killing-Feld der resultierenden Mannigfaltigkeit ist.

Beweis (durch Widerspruch im Fall der zeitähnlichen Killing Fields des Minkowski-Raums ... Lorentzianische Metrik). Annehmen, dass ϕ : M ' M ist eine Isometrie; wähle einen Punkt P W ¯ ; seit W Zeitlich ist immer ein Killing-Vektorfeld vorhanden K neben T P ( P ) M ' ; wähle etwas anderes Q W ¯ , dann entweder die Tangente an Q ist parallel zu K oder es ist nicht: wenn es nicht ist, W ¯ muss die Integralkurven von K schneiden und die Exzision bricht daher die Bijektion (durch Löschen von Bildpunkten) und es kann überhaupt keine Isometrie geben - ein Widerspruch. Daher W ¯ muss durch die Integralkurven von K regiert werden. (Muss wahrscheinlich für den allgemeinen Fall erweitert / umformuliert werden, da es keine Garantie dafür gibt, dass es irgendwo einen Killing-Vektor gibt, der die Exzisionsgrenze tangiert.)

Pädagogische Antworten sind doppelt willkommen - eine Antwort zu haben ist eine Sache, sie zu verstehen eine andere!

(mit geringfügigen Verbesserungen von math.se neu gepostet)

Antworten (1)

Es scheint mir, dass Ihre Frage nicht so viel mit Killing Fields zu tun hat. Es ist eine allgemeinere Frage. Betrachten Sie ein glattes Vektorfeld X über einen glatten (Hausdorff-)Verteiler M und nehmen Sie an, dass die Ein-Parameter-Gruppe lokaler Diffeomorphismen ϕ verbunden sein mit X ist global (was gleichbedeutend damit ist, das zu sagen X ist abgeschlossen ). Mit anderen Worten, wenn X M die Differentialgleichung

γ ˙ X ( T ) = X ( γ X ( T ) )
mit Anfangszustand
γ X ( 0 ) = X
eine (eindeutige) maximale Lösung zulässt γ X = γ X ( T ) für alle definiert T R .

Dafür gibt es hinreichende Bedingungen ϕ ist global (zum Beispiel passiert es vorausgesetzt M ist kompakt).

Hier entlang, ϕ : R × M ( T , X ) ϕ T ( X ) := γ X ( T ) M ist glatt und gut definiert. Darüber hinaus

(1) ϕ 0 = ich D

Und

(2) ϕ T ϕ τ = ϕ T + τ für jeden T , τ R .

Der Fall, den Sie in Betracht ziehen, erfordert dies auch M ist mit einer nicht entarteten Metrik ausgestattet G Und X ist ein komplett G -Killing-Vektorfeld.

In diesem Fall alle ϕ T : M M ist eine Isometrie.

Nun, um auf den allgemeinen Fall zurückzukommen, gilt der folgende Satz.

VORSCHLAG . Lassen A M eine offene Menge sein, deren Rand A ist eine glatte Kodimension- 1 eingebetteter Unterverteiler des glatten Verteilers M Und X ein glattes vollständiges Vektorfeld auf M . Dann sind die beiden folgenden Tatsachen äquivalent.

(A) ϕ T ( A ) = A Und ϕ T ( M A ¯ ) = M A ¯ für jeden T R .

(B) X ist tangential zu A .

Beweis .

(1) Wir beweisen, dass nicht (a) nicht (b) impliziert.

Wenn das falsch ist ϕ T ( A ) = A Und ϕ T ( M A ¯ ) = M A ¯ für alle T , dann muss es einen Punkt geben X 0 A so dass ϕ T 0 ( X 0 ) A oder ein Punkt X 0 M A ¯ so dass ϕ T 0 ( X 0 ) M A ¯ für einige T 0 R . Angenommen, ersteres ist gültig (letzteres kann ähnlich behandelt werden). Annehmen T 0 > 0 der andere Fall ist analog. Es gibt nun zwei Möglichkeiten für ϕ T 0 ( X 0 ) A . Einer ist ϕ T 0 ( X 0 ) A und in diesem Fall definieren S := T 0 . Die andere Möglichkeit ist ϕ T 0 ( X 0 ) M A ¯ . Definieren Sie in diesem Fall

S := sup { T [ 0 , + ) | ϕ τ ( X 0 ) A , τ < T } .
Diese Zahl existiert und ist endlich (weil die Menge nicht leer ist, da sie enthält 0 , Und T 0 < + ist eine Obergrenze), streng positiv, nicht größer als T 0 , und wieder ϕ S ( X 0 ) A .

(In der Tat, wenn ϕ S ( X 0 ) A Es gibt eine offene Nachbarschaft von ϕ S ( X 0 ) vollständig enthalten A so dass ϕ τ ( X 0 ) A auch für manche τ > S was für die Definition von unmöglich ist sup , Wenn ϕ S ( X 0 ) M A ¯ , da diese Menge offen ist, gäbe es eine offene Nachbarschaft von ϕ S ( X 0 ) vollständig enthalten M A ¯ so dass ϕ τ ( X 0 ) A in einigen ( S ϵ , S ] was wiederum für die Definition von unmöglich ist sup ; die einzige verbleibende Fall-ID ϕ S ( X 0 ) A .)

Lassen Sie uns beweisen, dass solche S (in beiden Möglichkeiten) kann nicht existieren, wenn (b) gilt. In der Tat, X | A ist ein wohldefiniertes glattes vollständiges Vektorfeld auf der glatten Mannigfaltigkeit A und damit ist das zugehörige Cauchy-Problem vorbei A mit Anfangszustand γ ˙ ( S ) = ϕ S ( X 0 ) A bei T = S gibt eine vollständige Lösung vollständig enthalten in A auch für T < S , aber diese Kurve jetzt als integrale Linie betrachtet X In M ist eindeutig bestimmt und wir wissen durch Hypothese, dass es bei beginnt X 0 A Widerspruch finden.

(2) Wir beweisen, dass nicht (b) nicht (a) impliziert.

Nehmen wir an, dass (b) falsch ist und stellen fest, dass (a) ebenfalls falsch ist. Gehe jetzt davon aus, dass es das gibt X 0 A so dass X ( X 0 ) steht quer dazu A . Als A ist eine eingebettete glatte Mannigfaltigkeit, X ist glatt und verschwindet nicht bei X 0 , ist es nicht allzu schwierig zu beweisen, dass es einen Koordinatenfleck gibt X 1 , X 2 , . . . , X N um X 0 In M ( N = D ich M ( M ) ) so dass X 0 ( 0 , 0 , , 0 ) , A ist der Teil des Flugzeugs X 1 = 0 im Bild des Diagramms enthalten, und die integralen Kurven von X sind die Kurven R T ( T , X 2 , , X N ) (siehe abschließendes ADDENDUM ). Da trennt sich das Flugzeug A aus M A ¯ , es ist offensichtlich, dass es Punkte gibt A in die eingezogen wird M A ¯ von ϕ und umgekehrt. Deshalb ϕ T ( A ) = A Und ϕ T ( M A ¯ ) = M A ¯ für jeden T R ist falsch.

QED

Offensichtlich, wenn X ein vollständiges Killing-Feld ist, betrifft das Ergebnis die zugehörige einparametrige Gruppe von Isometrien.


NACHTRAG . Das beweise ich hier

Lemma . Wenn S ist eingebettet N 1 -dimensionale glatte Untermannigfaltigkeit der N -dimensionale glatte Mannigfaltigkeit M , Und X ist ein glattes Vektorfeld vorbei M was bei nicht verschwindet X 0 S und ist nicht tangential (dh quer) zu S bei X 0 , dann gibt es einen Koordinaten-Patch X 1 , X 2 , . . . , X N um X 0 In M so dass X 0 ( 0 , 0 , , 0 ) , S ist der Teil des Flugzeugs X 1 = 0 im Bild des Diagramms enthalten, und die integralen Kurven von X sind (Einschränkungen um T = 0 von) die Kurven R T ( T , X 2 , , X N ) .

Beweis . Als S eingebettet ist, gibt es einen Koordinatenpatch ( U , ψ ) In M um X 0 S so dass ψ ( S U ) = { ( j 1 , , j N ) ψ ( U ) | j 1 = 0 } und wir können immer davon ausgehen ψ ( X 0 ) = ( 0 , , 0 ) . Jetzt X = A Y A j A ist so das Y 1 ( 0 , , 0 ) 0 nur weil X steht quer dazu S bei X 0 (die Koordinaten j 2 , , j N sind Koordinaten an S ). Die integralen Linien von X in Koordinaten befriedigen D j A D T = Y A ( j 1 ( T ) , , j N ( T ) ) . Wir sind frei zu beheben T = 0 genau an S für alle Kurven. Führen Sie nun die Koordinaten ein X 2 = j 2 , , X N = j N An S und schreibe die Integralkurven als glatte Funktionen j k = j k ( T , X 2 , , X N ) , Wo X 2 , , X N bezeichnet den Anfangspunkt an S (bei T = 0 ) der betrachteten Integralkurve. Die besagte Abbildung ist glatt, wie es aus Standardtheoremen über die glatte Abhängigkeit von Anfangsdaten von Cauchy-Problemen bekannt ist. Endlich definieren X 1 = T . Da die Jacobi-Matrix J = [ j A X B ] genau bei X 0 erfüllt

det J ( X 0 ) = j 1 T | X 0 = Y 1 ( 0 , , 0 ) 0
Das beweist Dinis Theorem X 1 = T , X B = j B Definieren Sie ein zulässiges glattes Koordinatensystem in M um X 0 . In lokalen Koordinaten X 1 , , X N , der Teil von S Die Eingabe des Bereichs der Koordinaten wird immer noch durch dargestellt X 1 = 0 (Weil X 1 = T und alle Integralkurven schneiden sich S bei T = 0 ) und lokal die Integralkurven von X sind trivial Einschränkungen herum T = 0 der Kurven R T ( T , X 2 , , X N ) .

Eine schnelle erste Bestätigung und ein großes Dankeschön! Muss das sorgfältig studieren, wird aber wahrscheinlich später akzeptieren.
Nur eine anfängliche Frage: Proposition (a) - Sie entfernen den Abschluss von (offen) A, ja? Lässt das M nicht ohne Grenzen? Ich wollte, dass A entfernt wird, nicht seine Grenze. Hinweis: Wenn Sie am Wochenende jederzeit im Chat verfügbar sind, lassen Sie es mich bitte wissen.
Ich entferne auch die Grenze. Das Bild ist so symmetrisch unter Vertauschung von A und M. Tut mir leid, ich bin zu beschäftigt und kann nicht chatten.
Ich habe noch ein paar Details hinzugefügt...
@Julian Moore Was ist mit den 50 Punkten des Kopfgeldes??
Obwohl ich noch ein paar Fragen habe, habe ich die Antwort akzeptiert - mit vielen Dank. Nach einigem Studium finde ich, dass die Antwort auf den Grundlagen der Lügentheorie basiert ... also habe ich eine Vorspeise, aber ich bin immer noch mathematisch unwissend, also (wenn Sie geneigt sind zu antworten) Q1: Wie überträgt sich der Beweis auf M \ A statt M\clos(A)? (Das ist der Schlüssel für mich und ich sehe nicht, wie die Symmetrie der Offenheit zwischen M & A hilft.) Und die 3 Zwischenstufen: "nicht schwer zu beweisen", "ist einfach", "trivial" - wo könnte ich finden ( in absteigender Reihenfolge der Wichtigkeit!) Weitere Hinweise zum Ausfüllen der Details? Nochmals mein aufrichtiges thx!
Kopfgeld? Ich dachte, das wäre vergeben, sobald ich angenommen hatte. Ich werde nachsehen, wenn ich nach Hause komme.
Ich habe in Meta danach gefragt; Obwohl es sein kann, dass Ihre Antwort direkt am Ende der Prämienperiode eingetroffen ist, betrachte ich den Fehler als meinen, da ich die Einzelheiten des Prämienprozesses nicht kenne, und werde als Gegenleistung eine weitere Prämie anbieten (wenn ich herausgefunden habe, wie es geht ) von 50, da mir das Ergebnis sehr wichtig ist und ich Ihre Antwort sehr schätze; Wenn man bedenkt, dass der Fehler bei mir liegt, ist dies völlig bedingungslos, aber jeder weitere Beitrag, den Sie anbieten, wird sehr geschätzt. Entschuldigung für die Unannehmlichkeiten. Ich habe auch gesehen, dass @Alfred-Centauri als Anerkennung ein weiteres Kopfgeld hinzugefügt hat!
Keine Sorge. Leider bin ich mit einigen Dingen ziemlich beschäftigt, um meiner Antwort weitere Details hinzuzufügen, ich werde es später (vielleicht am Wochenende) versuchen ...
OK @Julian Moore Ich habe die Zeit gefunden, einige weitere Beweise hinzuzufügen. Sag Bescheid ob es jetzt besser ist...
Sie sind sehr freundlich – und verständnisvoll in Bezug auf das Kopfgeldproblem; Ich bleibe aber dabei! Ihre Antwort ist in der Tat beispielhaft. Vielleicht bin ich jetzt in der Lage, meine Arbeit fertigzustellen!
Ich kann nur dann ein Kopfgeld hinzufügen, wenn das aktuelle Kopfgeld abgelaufen ist, und es muss das Doppelte sein, was ich gerne gebe.
Keine Probleme, ich habe endlich die 50 Punkte des Kopfgeldes (hinzugefügt von Alfred Centaury) erhalten, einer reicht!
Nichtsdestotrotz ... wie versprochen - und aus Prinzip und nach Treu und Glauben - habe ich mein eigenes Kopfgeld von 100 für Ihren beträchtlichen Einsatz hinzugefügt (mit Bitte , aber nicht bedingt durch Verlängerung auf A geschlossen ... reicht es zu sagen, dass es folgt für A geschlossen durch Symmetrie, da nicht darauf vertraut wird, zu welcher Menge die Grenze gehört??)
Bitte formulieren Sie Ihre Frage in einem klaren Formular und ich werde versuchen zu antworten.
Vielen Dank. Ich würde gerne wissen, wie der Beweis des Satzes verwendet werden kann, um zu erhalten: die Äquivalenz von: (a') ϕ T ( A ) = A Und ϕ T ( M A ) = M A für jeden T R , und (b), z A M eine offene Menge. Ich glaube , ich weiß, wie es geht, aber ich bin weit davon entfernt, zuversichtlich zu sein.
Es ist klar, dass (a) -> (a') weil, da A , M A ¯ , A sind disjunkte Mengen, deren Vereinigung s M , pro Unterschied (a) impliziert ϕ T ( A ) A und daher ϕ T ( A ) = A denn jeder ϕ T ist bijektiv. Nehmen wir nun an, dass (a') gilt. Wenn X ist nicht tangential an X 0 A , indem Sie das gleiche Argument ausnutzen, das ich in (2) verwendet habe, können Sie eine integrale Linie konstruieren, die ausgeht A vorbei für X ( X 0 ) . Also impliziert (a') insbesondere das X ist tangential zu A und so gilt (b), also (a')->(b) <->(a).
Zusammenfassend (a) -> (a') -> (b) -> (a) und so sind (a), (a') und (b) paarweise äquivalent.
In Bezug auf (a) -> (a') war ich etwas implizit. Genauer gesagt: Sobald Sie das wissen ϕ T ( A ) = A , aus der Hypothese (a) haben Sie auch ϕ T ( M A ) = ϕ T ( M A ¯ A ) = ϕ T ( M A ¯ ) ϕ T ( A ) = M A ¯ A = M A . Dies vervollständigt den Beweis, dass (a) -> (a').
Das dachte ich mir ;) Schön gemacht - und noch einmal vielen Dank!
"aus Standardsätzen über die glatte Abhängigkeit von Anfangsdaten von Cauchy-Problemen bekannt". Ich würde es vorziehen, spezifische Theoreme mit direkter Anwendung zu zitieren. Oder zumindest zu wissen, was auf diesem Niveau als "bekannt" gilt ... (würde Cauchy-Kovalevski einer von ihnen sein?) Können Sie mit einigen Einzelheiten helfen?
Nun, es ist nichts anderes als der Satz, den ich bereits verwendet habe, um ihn zu definieren ϕ T ( X ) . Die Lösung einer Differentialgleichung erster Ordnung, die durch ein glattes Vektorfeld auf einer glatten Mannigfaltigkeit definiert ist, ist eine gemeinsam glatte Funktion des Parameters entlang Lösungen und Anfangsbedingung.
Hallo Valter - eine nicht-technische Frage, wenn ich darf; Offensichtlich hat die ursprüngliche Behauptung funktioniert, aber ich bin neugierig, meine "Intuition" zu beurteilen. Würden Sie sagen, dass die Beweisskizze auch gut motiviert war oder wichtige Faktoren übersehen hat? (FYI, Anwendung auf eine Mannigfaltigkeit M: Das Platzieren einer Lorentz-Signaturmetrik auf M kompakt ist äquivalent (siehe z. B. Geroch J. Math. Phys '67 für Details) zum Platzieren eines kontinuierlichen Vektorfelds auf M, also für M ein raumartig begrenzter Abschnitt von Minkowski-Raumzeit, der Satz ist direkt anwendbar, wie ich gehofft hatte!)
Hallo Julian, es war nicht nur ein Kommentar, ich wollte deine ursprüngliche Frage gut verstehen. Aber ja, ich denke, Ihre allgemeine Idee war richtig, tatsächlich habe ich nichts anderes getan, als Ihre Idee in einen allgemeineren Kontext zu erweitern. (Ich schreibe von meinem Handy, ich hoffe, es ist verständlich).
Hallo Valter, auch am Telefon und ein wohlverdientes Bier genießend! Ihr früherer Kommentar scheint verschwunden zu sein, aber es lohnt sich zu sagen, dass sich die "Erweiterung" gelohnt hat. Hinweis: Ich habe den Parameter nicht erwähnt, weil ich 1-Parameter-Diffeomorphismen als solche nicht kannte ;) Ich neige dazu, nur dann etwas über solche Dinge zu lernen, wenn ich einen Bedarf dafür finde, wie in diesem Fall ;)
Beim LaTeXing & Reorganisieren des Beweises ... "Sei A⊂M eine offene Menge, deren Rand ∂A ..." Muss das umformuliert werden? Die Grenze einer offenen Menge ist die leere Menge, nicht wahr? Sollte es sich auf die Schließung beziehen? Danke!
Die Grenze (oder Grenze) des offenen Balls { X R N | | | X | | < 1 } Ist { X R N | | | X | | = 1 } und nicht die leere Menge. Die Grenze einer Menge darf keinen Schnittpunkt mit der Menge selbst haben.
Ich habe die falsche Aussage gelesen, die ich irgendwo scheinbar maßgeblich zitiert habe, aber ich vertraue Ihnen mehr als "dem Internet";) Danke
Sie haben wahrscheinlich die Definition von "Mannigfaltigkeit mit Grenze" gefunden, bei der die Grenze per Definition Teil der Menge ist und ein präzises Zusammenspiel (nicht nur topologisch) zwischen der Masse und der Grenze besteht. Ich habe stattdessen die allgemeine topologische Definition der Grenze verwendet, die zur Lösung Ihres Problems völlig ausreicht.